Difference between revisions of "2024 AMC 10A Problems/Problem 2"

(Video Solution)
 
(One intermediate revision by the same user not shown)
Line 2: Line 2:
  
 
<math>\textbf{(A) } \text{R E D} \qquad\textbf{(B) } \text{L S T E R} \qquad\textbf{(C) } \text{Q E D} \qquad\textbf{(D) } \text{L M N O P} \qquad\textbf{(E) } 42 </math>
 
<math>\textbf{(A) } \text{R E D} \qquad\textbf{(B) } \text{L S T E R} \qquad\textbf{(C) } \text{Q E D} \qquad\textbf{(D) } \text{L M N O P} \qquad\textbf{(E) } 42 </math>
 +
 +
==Video Solution==
 +
https://youtu.be/sHQgxWPJSKc?si=bTNk8EXGJw5pThy7

Latest revision as of 01:43, 3 August 2024